Find the midpoint of the segment with the following endpoints.
(9, 2) and (5,5)

Answers

Answer 1

Answer:

(7,3.5)

Step-by-step explanation:

Midpoints

x = (x1 + x2)/2 = (9 + 5)/2 = 14/2 = 7

y = (y1 + y2)/2 = (2 + 5)/2 = 7/2 = 3.5


Related Questions

if 2 shmoop interns can give 10 total foot massage in an hour, how many foot massages can 5 shmoop interns give in an hour?

Answers

1 shmoop intern can give 5 foot massages in an hour, so 5 smhoop interns could give 25 foot massages in an hour.

What does congruent mean? Select all that apply. PLEASE HELP!

A None of the above
B Same Shape
C Corresponding angles are the same measure
D Corresponding sides are the same length

Answers

Answer C is the correct choice.

A skateboard travels 55 feet in 4 seconds. Is the skateboard going faster,
slower, or the same speed as the scooter?

Answers

Answer:

Slower

Step-by-step explanation

The scooter travels 60 feet in 4 seconds, so it is going faster than the skateboard, which travels 55 feet in 4 seconds. A cargo ship traveled 150 nautical miles in 6 hours at a constant speed. How far did the cargo ship travel in one hour?

Please help will give crown! Here is the question 2(-2)-4=12 solve it and tell me where it goes on a graph if you want, I'll give 400 points, if i can!

Answers

The solution to the given equation (2(-2) - 4 = 12) is -20 and its graphical solution is shown in the image attached below.

What is a number line?

A number line can be defined as a type of graph with a graduated straight line which contains both positive and negative numerical values that are placed at equal intervals along its length.

Next, we would solve the given equation as follows:

2(-2) - 4 = 12

Let us introduce a variable x into the equation as follows:

x = 2(-2) - 4 - 12

x = -4 - 4 - 12

x = -20

Read more on number line here: brainly.com/question/16614696

#SPJ1

A person on a moving walkway travels 80 ft in 40 sec. What equation represents this relationship? How many feet will the person travel in 10 sec?A. y = 1/2x; 5 ftB. y = 4x; 40 ftC. y = 2x; 20 ftD. y = 2x; 2 ft

Answers

We need to find which expression represents the situation given, in this case we have two values a distance (80 ft) and the elapsed time (40 s). The time is the input of the equation, it will be placed on the "x" variable and the distance is the output of the equation, it will be placed on the "y" variable. We need to find the expression that outputs "80" when we input 40. Let's check them one by one:

A.

[tex]y=\frac{1}{2}\cdot40=20[/tex]

This outputs 20 ft when we input 40 s, therefore it is not this one.

B.

[tex]y=4\cdot40=160[/tex]

This one outputs 160 ft when we input 40 s, therefore it is not this one.

C.

[tex]y=2x=2\cdot40=80\text{ }[/tex]

This one outputs 80 ft when we input 40 s, therefore this is the correct expression.

We now need to use this expression to calculate the distance the person will travel in 10 seconds, so we will make x equal to 10 s.

[tex]y=2\cdot10=20\text{ }[/tex]

The person will walk 20 ft.

The correct option is "C".

Round to the place value of the underlined digit 698,072 the underlined digit is 6

Answers

We get 700,000 after rounding off to the place value of the underlined digit in the number 698,072.

According to the question,

We have the following number with 6 as the underlined digit:

698,072

When rounding off the number, please note that the digits to the right of the underlined digit would be zero. However, we need to be careful where we need to scale up the underlined number or it would remain the same.

For this, we need to check the number to the right of the underlined digit. If the number right next to the underlined digit is greater than 5 then the underlined digit has to be scaled up. And if it is less than 5 then the underlined digit remains the same.

So, after rounding off we will get:

700,000

Hence, the number after rounding off to 6 in 698,072 is 700,000.

To know more about place value here

https://brainly.com/question/27734142

#SPJ1

Solve for y in terms of u, v, w, and x.
vw = -yux
VW
y =

Answers

Answer:

y= vw/ux

Step-by-step explanation:

hope this help

What is the distance between points A(2,9) and B(-2,6)? Round to the nearest whole number.

Answers

The distance between the point is 5.

The formula to find the distance between two points (x1, y1) and (x2, y2)

distance = [tex]\sqrt{(x2-x1)^{2} + (\\ y2-y1)^{2} }[/tex]

Substituting points A(2,9) and B(-2,6), we get

distance = [tex]\sqrt{(-2-2)^{2} + ( 6-9)^{2} }[/tex]

              =[tex]\sqrt{(-4)^{2} + (-3)^{2} }[/tex]

              = [tex]\sqrt{16 + 9}[/tex]

              =[tex]\sqrt{25}[/tex]

             = 5

Therefore the distance between the two points is 5.

To know more about the distance between the two points refer to the link given below:

https://brainly.com/question/7243416

#SPJ1

Use the confidence level and sample data to find a confidence interval for estimating the population μ. Round your answer to the same number of decimal places as the sample mean.

A group of 59 randomly selected students have a mean score of 29.5 with a standard deviation of 5.2 on a placement test. What is the 90% confidence interval for the mean score, μ, of all students taking the test?

Answers

The 90% confidence interval for the mean score, μ, of all students taking the test is; CI = (28.36, 30.64)

What is the Confidence Interval?

The formula for confidence interval is;

CI = x' ± z(s/√n)

where;

CI = Confidence Interval

x' is sample mean

z is critical value at confidence level

s is standard deviation

n is sample size

We are given;

n = 59

x' = 29.5

s = 5.2

z at CL 0f 90% = 1.645

Thus;

CI = 29.5 ± 1.645(5.2/√59)

CI = 29.5 ± 1.136

CI = (28.36, 30.64)

Read more about Confidence Interval at; https://brainly.com/question/17097944

#SPJ1

Find all angles, osO<360, that satisfy the equation below, to thenearest 10th of a degree.cos(0) =-1/6

Answers

The cosine of the angle given is negative.

Cosine is negative in the second and third quadrant

[tex]\begin{gathered} 0\leq x\leq360 \\ \cos (\theta)=-\frac{1}{6} \\ \theta=\cos ^{-1}(\frac{1}{6}) \\ \theta=80.41^0 \\ \theta=80.4^0\text{ (to the nearest tenth)} \\ \end{gathered}[/tex]

In the second quadrant,

[tex]\begin{gathered} 180-\theta \\ 180-80.4=99.6 \\ \theta=99.6^0 \end{gathered}[/tex]

In the third quadrant,

[tex]\begin{gathered} 180+\theta \\ 180+80.4=260.4 \\ \theta=260.4^0 \end{gathered}[/tex]

Therefore, the values that satisfy the equation are 99.6 and 260.4 degrees.

Convert 32.5% to a decimal number. If needed round to the nearest thousandth.

Answers

In order to convert a percent to a decimal number we need to move the decimal point two units to the left

[tex]32.5\text{\%}=0.325[/tex]

a triangle has two sides of 10 and 6 with an area of 12 units. what is the missing length.

Answers

A triangle that has two sides of 10 and 6 with an area of 12 units will have the third side length of 4.

What is the area of a triangle?

The triangle can be described as the the figure that has threes sides which can be equal or varies base on the type of the triangle it is, and the area of the triangle is given, we can use the formula for the are of triangle which i s (1/2  * b * h )

Then the Area of a triangle = (1/2  * b * h)

we were given the two sides already with the area , then we can input the values as :

12 =( 1/2 * 6 * x)

where x is the third sides that is needed

x = 4

Read more about triangle here:

https://brainly.com/question/17335144

#SPJ1

Which number line shows the solution to the inequality 5 ≤ 8-3x ≤ 14?

Answers

The number line 5 ≤ 8-3x  shows the solution to the inequality as x cannot be more than 1.

What is inequality?

A inequality compares two values and indicates whether one is smaller, larger, or simply not equal to the other.

a  ≠ b is the information that a and b are not equal.

If a  > b, a is greater than b. (these two are defined as strict inequality)

a ≤ b proves that a is less than or equal to b

a ≥ b proves that a is greater than or equal to b

The number line 5 ≤ 8-3x  shows the solution to the inequality as x cannot be more than 1.

If x is more than 1, lets say 2

Then the inequality as follows

5 ≤ 8-3(2) ≤ 14

5 ≤ 8-6 ≤ 14

5 ≤ 2 ≤ 14  this statement becomes false

Therefore x, must be lower than 1

Learn more about inequality

https://brainly.com/question/24372553

#SPJ10

How many solutions over the complex number system does this polynomial
have?
3x6x³-4x²+3x+52 = 0
Enter your answer as an integer.

Answers

Polynomials are algebraic expressions that consist of the variables and coefficients. Six complex number system, polynomial 3x6x³-4x²+3x+52 = 0 have.

What do you mean by polynomial?

Polynomials are algebraic expressions that consist of the variables and coefficients. Variables are also sometimes called the indeterminates. We can perform the arithmetic operations such as addition, subtraction, multiplication, and also positive integer exponents for polynomial expressions but not division by variable.

We have following polynomial for this situation.

We observe that polynomial graph only intersects the vertical axis once.

There are no additional x-axis cutoff points in graph.

As a result, polynomial:

6 difficult solutions

No actual solution

So, we can 6 difficult solutions.

We have following polynomial for this situation.

We observe that polynomial graph only intersects the vertical axis once.

There are no additional x-axis cutoff points in graph.

As a result, polynomial:

6 difficult solutions

No actual solution

So, 6 complex solutions can be said to exist.

To know more about polynomial visit: https://brainly.com/question/11536910

#SPJ9

Drag and drop the number to match the division problem to its quotient.

Answers

Answer:

The correct number is -27.

Step-by-step explanation:

-81 ÷ 3 = -27

help me please!!!!!!11!

Answers

Answer:

[tex]y=x^2-4x-2[/tex]

Step-by-step explanation:

Standard Form of Quadratic:

[tex]y=ax^2+bx+c[/tex]

Vertex Form of Quadratic:

[tex]y=a(x-h)^2+k[/tex]

h = vertex x

k = vertex y

Think of h and k as the ordered pair (h,k).

Match this ordered pair to the vertex ordered pair.

(h,k) = (2, -6)

Plug in (h,k) into vertex form:

[tex]y = a(x-2)^2-6[/tex]

Use other point to solve for a.

[tex]3 = a(5-2)^2-6[/tex]

[tex]3=a(3)^2-6[/tex]

[tex]3=9a-6[/tex]

[tex]9=9a[/tex]

[tex]a=1[/tex]

Plug a back into vertex form and expand the expression into standard form:

[tex]y=(x-2)^2-6[/tex]

[tex]y=(x-2)(x-2)-6[/tex]

Use F.O.I.L. method to expand (x-2)(x-2).

F(first):  [tex]x*x=x^2[/tex]

O(outer):  [tex]x*-2=-2x[/tex]

I(inner): [tex]-2*x =-2x[/tex]

L(last): [tex]-2*-2=4[/tex]

[tex]y=x^2-2x-2x+4-6[/tex]

Combine like terms:

[tex]y=x^2-4x-2[/tex]

29-(11-2³)+6²÷4 simplify each Expression

Answers

The simplified form of the expression 29 - ( 11 - 2³ ) + 6² ÷ 4 is 35

What is the simplified form of the given expression?

Given the expression in the question;

29 - ( 11 - 2³ ) + 6² ÷ 4

To simply, replace 2 raised to the power of 3 with 8 and 6 raised to the power of 2 by 36

29 - ( 11 - 2³ ) + 6² ÷ 4

29 - ( 11 - 8 ) + 36 ÷ 4

Now, remove the parenthesis by performing the operation inside the parenthesis.

29 - ( 11 - 8 ) + 36 ÷ 4

29 - 3 + 36 ÷ 4

Now, divide 36 by 4

29 - 3 + 36 ÷ 4

29 - 3 + 9

Next, add -3 and 9

29 + 6

Add 29 and 6

35

Therefore, 35 is the simplified form of the expression.

Learn more about FOIL and PEMDAS here: https://brainly.com/question/36185

#SPJ1

To find the height of a very tall pine tree, you place a mirror
on the ground and stand where you can see the top of the
pine tree. How tall is the tree?
6 ft
2 ft 24 ft
ти
How tall is the tree

Answers

If to find the height of a very tall pine tree, you place a mirror on the ground and stand where you can see the top of the pine tree. The height of  the tree is 72 feet.

Height of a tree

Given data;

6 ft

2 ft 24 ft

Hence,

Angle of incidence = Angle of reflection

Hence,

m ∠ ACD = m ∠ ECD

Two triangle will only be the same if their corresponding angles are congruent
So,

Sine m ∠ ACD = m ∠ ECD

m Δ ABC  = Δ EDC

The corresponding of similar triangles are proportional

Hence,

BC / DC = AB / ED

= 2 / 24 = 6 / ED

2ED = 24 × 6

2ED = 144

Divide both side by 2

ED = 144 / 2

ED = 72 feet

Therefore we can conclude that the tree height is measure at 72 feet.

Learn more about height here:https://brainly.com/question/73194

#SPJ1

Triangle A has a height of 8 inches and a base of 2 inches. Triangle B has a height of 16 inches, what is the base?

Answers

The base of Triangle B is 1 inches.

Given,

Triangle A has a height of 8 inches

and base of Triangle is 2 inches

Let's Find the Area of Triangle A

We know that the formula of Area of Triangle

Area of Triangle = [tex]\frac{1}{2}[/tex] × b × h

where, b is the base and h is the height of triangle.

Plug the values of base and height in above formula.

Area of Triangle A  = [tex]\frac{1}{2}[/tex] × 2 × 8

Hence, Area of Triangle A is 8 sq. inches

In this question, Some part is missing that is:

Triangle A is similar to Triangle B

Now, For Triangle B is given:

Triangle B has a height of 16 inches

and, to find the base of the triangle.

We know, Area of Triangle A is 8 sq. inches.

Area of Triangle B =  [tex]\frac{1}{2}[/tex] × b × h

8 =  [tex]\frac{1}{2}[/tex] × b × 16

8 = 8b

b = 1 inches

Hence, The base of Triangle B is 1 inches.

Learn more about Area of Triangle at:

https://brainly.com/question/19305981

#SPJ1

Write the equation of the line in slope-intercept form.

The line is parallel to y + x = 3 and passes through the point (-12, 0).

Please help i will give big points

Answers

Answer:

The line is parallel to y + x =3

=> y = -x+3

the slope = -1

passes (-12, 0)

the equation would be :

y-0 = -1(x+12)

y = -x -12 Y=-x+12

Turn into mx+b form.

Y=-x+3

Plug in (-12,0)

0=-12+b

b=12

y=-x+12

It is negative X, because the slope has to be the same since it is paralle

Step-by-step explanation:

Algebra 2, 50 points, include steps please
(6-4i)(1+5i)-(3-i)

Answers

The simplified representation of the complex number expression is

23 + 37i

a + bi, where a and b are real numbers, can be used to represent any complex number.

A complex number is a component of a number system that includes an element with the symbol I, sometimes known as the imaginary unit, and that extends the real numbers by satisfying the equation i² = -1. i was described as an imaginary number by René Descartes since no real number can fulfil the aforementioned equation. The complex number a + bi is known as having real and imaginary parts, respectively, A and b. The group of complex numbers is denoted by the letter C.

The complex number expression is (6-4i)(1+5i)-(3-i)

now let us simplify,

or, (6-4i)(1+5i)-(3-i)

or, 6 + 30i - 4i -20i² -3 + i

or, 6 + 36i + 20 - 3 + i (we know that i² = -1)

or, 23 + 37i

Therefore the simplified complex expression is 23 + 37i

To learn more about complex numbers visit:

https://brainly.com/question/20566728

#SPJ1

The degree of the polynomial function f(x) is 3.
The roots of the equation f(x) = 0 are -1, 0, and 4.
Which graph could be the graph of f(x)?

Answers

The graph of the polynomial function x³ - 3x² - 4x = 0 is shown if figure.

What is Polynomial function?

A mathematical expression of algebraic terms each of which consists of a constant multiplied by one or more variables raised to a nonnegative integral power.

Given that;

The degree of the polynomial function f(x) is 3.

And, The roots of the equation f(x) = 0 are -1, 0, and 4.

Now, The polynomial function are calculated as;

f (x) = 0

(x + 1) (x - 0) (x - 4) = 0

x (x + 1) (x - 4) = 0

x (x² - 4x + x - 4) = 0

x (x² - 3x - 4) = 0

x³ - 3x² - 4x = 0

Thus, The polynomial function is;

x³ - 3x² - 4x = 0

Therefore, The graph of the polynomial function x³ - 3x² - 4x = 0 is shown if figure.

Learn more about the polynomial function visit:

https://brainly.com/question/7693326

#SPJ1

The volleyball team bought 12 new volleyballs. The price for each volleyball was $26. How much did the team spend? Select the choice that shows all of the partial products of 12 × 26.

A.
12, 70, 4, 400

B.
12, 90, 14, 200

C.
12, 62, 4, 260

D.
12, 60, 40, 200

Answers

Answer:  Choice D)  12, 60, 40, 200

========================================================

Explanation:

Imagine a rectangle that is 12 by 26. Multiplying the length and width will get us the area.

We can break the 12 into 10+2

Also, break the 26 into 20+6

Check out the diagram below. We have 4 smaller rectangles. Finding the product of those sub-areas will then get us the partial products needed.

In the bottom right corner we have a 2 by 6 rectangle, so it has area 2*6 = 12 square units. This is one of the four partial products. The other partial products are found the same way.

Once you determine the four partial products, we then add them up

12+60+40+200 = 12+100+200 = 12+300 = 312

Therefore, 12*26 = 312 which we can use a calculator to confirm.

Simplify the quantity 8 minus one third times the square root of 9 end quantity squared plus the quantity 1 minus 5 end quantity squared.

Answers

The statement quantity 8 minus one third times the square root of 9 end quantity squared plus the quantity 1 minus 5 end quantity squared is simplified to 65

What are fractions?

Fractions are defined as parts of a whole element, set or number.

There are various types of fractions, which includes;

Complex fractionsProper fractionsSimple fractionsMixed fractionsImproper fractions

Based on the information provided, we have;

8 minus one third times the square root of 9 = 8 - 1/3(√9)1 minus 5 end quantity squared = (1 - 5)²

Now, substitute the values

(8 - 1/3(√9))² +  (1 - 5)²

Find the square root

(8 - 1/3(3))²+ (-4)²

Find the square

(8 - 1)² + 16

7² + 16

49 + 16

Add the values

65

Hence, the value of the expression is 65

Learn more about square root here:

https://brainly.com/question/428672

#SPJ1

what operation would be completed second in the following expression

Answers

In the PEMDAS method of solving , it goes to the five basic operations

1.Parenthesis 2. Exponents 3. Multiplication 4. Addition 5.Substraction

In this exercise there are NOT parenthesis, so then the first operation is Exponentiation and 2nd operation is Multiplication

So the answer is Multiply, the first option.

The probability that a new machine will not need any repairs within t years from now is modeled by an exponential function of t. This probability is multiplied by 0.2 whenever the time period t is extended by 3 years as shown by the function belowf(t) = (0.2)^t/3 If the probability that the machine does not need repairs right now is 1, what is the probability that the machine will not need repairs within 12 years from now, according to the model? 0.8 0.05 0.008 0.0016

Answers

viven

t = time

function

[tex]f(t)=0.2^{t/3}[/tex]

what is the probability that the machine will not need repairs within 12

rocedure

[tex]\begin{gathered} f(12)=0.2^{12/3} \\ f(12)=0.0016 \end{gathered}[/tex]

The probability would be 0.0016

What is the answer to the question

Answers

From the right triangle, and using the definitions of the tangent and the cosine functions, we have:

[tex]\begin{gathered} \tan B=\frac{AC}{BC}=\frac{b}{a}\Rightarrow b=a\cdot\tan B...(1) \\ \\ \cos B=\frac{BC}{AB}=\frac{a}{c}\Rightarrow c=\frac{a}{\cos B}...(2) \end{gathered}[/tex]

From the problem, we identify:

[tex]\begin{gathered} B=55.7\degree \\ a=266\text{ km} \end{gathered}[/tex]

Finally, using these values, we can find b and c.

Using (1):

[tex]\begin{gathered} b=266\cdot\tan55.7\degree \\ \\ \therefore b=389.941\text{ km} \end{gathered}[/tex]

Using (2):

[tex]\begin{gathered} c=\frac{266}{\cos55.7\degree} \\ \\ \therefore c=472.028\text{ km} \end{gathered}[/tex]

what is the standard algorithm for 72 / 3​

Answers

Long division gives the answer of 24 when 72 is divided by 3.

What is long division calculator?

Long division in mathematics is a strategy for breaking down complicated division problems into a series of simpler steps. It is the approach that division-based issues are typically solved using. The dividend, the quotient, the remainder, and the divisor can all be seen in the following long division.

So as we know:

Start by arranging it so that the dividend 72 is on the right side and the divisor 3 is on the left:

 3 ⟌ 7 2  

Start by arranging it so that the dividend 72 is on the right side and the divisor 3 is on the left:

    2    

   3 ⟌ 7 2  

Write the result (3 x 2 = 6) underneath the dividend after multiplying the divisor by the outcome from the previous step.

     2    

   3 ⟌ 7 2  

       6    

Write the solution below after subtracting the result from the previous step from the dividend's first digit (7 - 6 = 1).

Reduce the dividend's second digit (2) as follows:

    2    

   3 ⟌ 7 2  

     - 6    

       1 2  

The bottom number (12) is multiplied by the divisor (3) four times (s). Put 4 thus on top

Subtract the result of the previous step (3 x 4) from the divisor to get the solution (12), which you should write at the bottom:

  2 4  

   3 ⟌ 7 2  

     - 6    

       1 2  

      1 2  

Take the number above it and deduct the outcome from the preceding calculation. Write the response at the bottom after calculating (12 - 12 = 0).

The top number is the solution, and the remaining value is the bottom one.

Therefore, the result of 72 divided by 3 using long division is 24.

To know more about long division method , click here:
https://brainly.com/question/28784606

#SPJ13

From a window 100 ft above the ground in building A, the top and bottom of building B are sighted so that the angels are 70 degrees and 30 degrees respectively. Find the height of building B?

Answers

Let's begin solving the problem by illustrating the problem using a diagram:

Let the height of building B be x

Re-drawing the triangles to show the unknown side:

Using trigonometric ratios and sine rule

Hence:

[tex]\begin{gathered} \frac{\sin110}{\frac{100}{\cos30}}\text{ = }\frac{\sin 40}{x} \\ \text{Cross}-\text{Multipy} \\ \text{x }\times\text{ sin110 }=\text{ sin40 }\times115.47 \\ \text{Divide both sides by sin110} \\ \text{x = }\frac{\sin 40\text{ }\times\text{ 115.47}}{\sin \text{ 110}} \\ x\text{ = 78.986} \\ x\text{ }\approx\text{ 79 ft} \end{gathered}[/tex]

The height of the building B is 79 ft

What is the correct answer

Answers

The measure of angle B in triangle ABC is 158°.

What is meant by the triangle?

A triangle is a 2-dimensional closed shape with three sides, three angles, and three vertices. A triangle would be a category of polygon.

The sum of a triangle's three interior angles is always 180.The sum of any two triangle sides is always bigger than this same length of a third side.A triangle's area is equal to half the product of it's own base and height.

For the given angles of the triangle ABC.

∠BAC = 78° , ∠CAD = 46° and ∠ADB = 110°.

Now,

∠BAC = ∠BAD + ∠DAC

Put the values;

78° = ∠BAD + 46°

∠BAD = 78° - 46°

∠BAD = 32°

Now, in a triangle; the sum of all three angles is 180°.

In triangle ADB

∠ABD + ∠BAD + ∠BDA = 180°

Put the values;

∠ABD +  32° +  110°  =  180°

∠ABD = 180°  -  142°

∠ABD = 158°

Thus, the measure of angle B is 158°.

To know more about the triangle, here

https://brainly.com/question/17335144

#SPJ13

Other Questions
The Rwandan genocide was a result ofan economic crisis resulting from corruption and failed crops. ethnic tensions that had their roots in the period of Belgian rule.election fraud that occurred soon after independence from Belgium. conflict between two military leaders over control of the oil industry. Problems with communications are always the fault of the speaker (sender). Question 12 options: True False Match the trigonometry formula to its correct ratio. Reduce the ratio if necessary. Choose the correct use of the Spanish preposition para in the following sentence.Para aprender un idioma bien, hay que practicar fuera del saln de clases.intended for a goal or destination"by," referring to a deadline"in order to," followed by an infinitive"to," referring to a destination Which of the following would be considered an aspiration of the Antebellum reform movements?Group of answer choicesWestward expansionManifest DestinyEconomic growthMoral Improvement 10. A blood alcohol content (BAC) of 0.08 g/100 mL means (3x+4x x + 7) (x + 2) If a company is using a cost-focused pricing strategy, it will set prices for products based on __________________.A.) What they think customers will likeB.) How much money they want to make C.) how much it costs to make themD.)the current economy Why are hydrophilic/polar substances unable to travel easily through the membrane Blake was assigned the problem below. Analyze his work and determine if it is correct.If it is wrong, describe all mistakes.Problem:53x-5 = ( 625 )#*?53x-s5 "(x+2)3x-5 = 4 (x+7)3x-5= 4x+7-3x-3xS = x +7-7-7x= -12 Light travels 1.8x 10^7 kilometers in one minute. How far does it travel in 8 minutes circle C has centre D and equationx^2+y^2+2x-8y+8=0A line is drawn through the point P (4,6),so that it touches the circle C at the point TShow that PT=square root 20find the equation of the circle centre P which passes through the point T is part of the marketing mix where the methods of providing a service is used.O Physical evidenceProcessO PlaceO Promotion Given the following Information, find the length of the missing side. Leave your answer as a simplified radical.Hypotenuse = 10Long = ?A. 5B. 5/3C. 10D. 10/3 tyrone and his partner, live with their young son, as well as tyrone's 25-year-old brother, traevon. this example illustrates a(n) family structure. A garden table and a bench cost $830 combined. The garden table costs $80 more than the bench. What is the cost of the bench? A 32 ounce box of Fruit Loops cost $3.99 and a 24 ounce box of Honey Nut Cheerios cost $3.29.Which is the better deal and why? What is the price of each Why would you take an antihistamine?a.) to cure an allergy b.) to relieve allergy symptoms c.) to allow an allergic reaction d.) to build immunity Write a summary about the relative positions of the sun, earth and moon throughout the lunar cycle. Your summary must should be a minimum of 5 sentences. find the value of k given both points lie on a line passing through the orgin (-12,-2) and (k,8)